Difference between revisions of "2018 AMC 8 Problems/Problem 21"

(Problem 21)
m
Line 6: Line 6:
 
==Solution==
 
==Solution==
  
Looking at the values, we notice that <math>11-7=4</math>, <math>9-5=4</math> and <math>6-2=4</math>. This means we are looking for a value that is four less than a mulitple of 11, 9 and 6. The least common multiple of these number is <math>11*3^2*2=198</math>, so the numbers that fulfill this can be written as <math>198k-4</math>, where k is a positive integer.  This value is only a three digit integer when <math>k</math> is <math>1, 2, 3, 4</math> or <math>5</math>, which output <math>194, 392, 590, 788,</math> and <math>986</math> respectively. Thus we have 5 values, so our answer is <math>\boxed{E}</math>
+
Looking at the values, we notice that <math>11-7=4</math>, <math>9-5=4</math> and <math>6-2=4</math>. This means we are looking for a value that is four less than a multiple of <math>11</math>, <math>9</math>, and <math>6</math>. The least common multiple of these number is <math>11*3^2*2=198</math>, so the numbers that fulfill this can be written as <math>198k-4</math>, where k is a positive integer.  This value is only a three digit integer when <math>k</math> is <math>1, 2, 3, 4</math> or <math>5</math>, which gives <math>194, 392, 590, 788,</math> and <math>986</math> respectively. Thus we have <math>5</math> values, so our answer is <math>\boxed{\textbf{(E) }5}</math>
  
 +
==See Also==
 
{{AMC8 box|year=2018|num-b=20|num-a=22}}
 
{{AMC8 box|year=2018|num-b=20|num-a=22}}
  
 
{{MAA Notice}}
 
{{MAA Notice}}

Revision as of 16:58, 21 November 2018

Problem 21

How many positive three-digit integers have a remainder of 2 when divided by 6, a remainder of 5 when divided by 9, and a remainder of 7 when divided by 11?

$\textbf{(A) }1\qquad\textbf{(B) }2\qquad\textbf{(C) }3\qquad\textbf{(D) }4\qquad \textbf{(E) }5$

Solution

Looking at the values, we notice that $11-7=4$, $9-5=4$ and $6-2=4$. This means we are looking for a value that is four less than a multiple of $11$, $9$, and $6$. The least common multiple of these number is $11*3^2*2=198$, so the numbers that fulfill this can be written as $198k-4$, where k is a positive integer. This value is only a three digit integer when $k$ is $1, 2, 3, 4$ or $5$, which gives $194, 392, 590, 788,$ and $986$ respectively. Thus we have $5$ values, so our answer is $\boxed{\textbf{(E) }5}$

See Also

2018 AMC 8 (ProblemsAnswer KeyResources)
Preceded by
Problem 20
Followed by
Problem 22
1 2 3 4 5 6 7 8 9 10 11 12 13 14 15 16 17 18 19 20 21 22 23 24 25
All AJHSME/AMC 8 Problems and Solutions

The problems on this page are copyrighted by the Mathematical Association of America's American Mathematics Competitions. AMC logo.png